K
Khách

Hãy nhập câu hỏi của bạn vào đây, nếu là tài khoản VIP, bạn sẽ được ưu tiên trả lời.

11 tháng 1 2018

17n; 17n+1; 17n+2 là 3 số nguyên liên tiếp nên có đúng một số chia hết cho 3 
* nếu n chia hết cho 3 => 17n chia hết cho 3 => (17n+1) và (17n+2) đều không chia hết cho 3, mà 3 là số nguyên tố => (17n+1)(17n+2) không chia hết cho 3 

* 17 và 3 là hai số nguyên tố cùng nhau nên nếu n không chi hết cho 3 thì 17n cũng không chia hết cho 3 => (17n+1) hoặc (17n+2) có một số chia hết cho 3 
=> (17n+1)(17n+2) chia hết cho 3 

Tóm lại: (17n+1)(17n+2) chia hết cho 3 khi và chỉ khi n không chia hết cho 3 
------------------------------ 
Giải xong câu 2 là hiểu ngay bạn ghi đó là các số mủ 
17ⁿ, 17ⁿ+1 và 17ⁿ+2 là 3 số tự nhiên liên tiếp, nên có một số chia hết cho 3, mà 17ⁿ không chia hết cho 3, nên một trong hai số 17ⁿ+1 hoặc 17ⁿ+2 chia hết cho 3 

=> (17ⁿ+1)(17ⁿ+2) chia hết cho 3 

11 tháng 1 2018

Xét : 17^n.(17^n+1).(17^n+2)

Ta thấy 17^n;17^n+1;17^n+2 là 3 số nguyên liên tiếp nên có 1 số chia hết cho 3

=> 17^n.(17^n+1).(17^n+2) chia hết cho 3

=> (17^n+1).(17^n+2) chia hết cho 3 ( vì 17^n ko chia hết cho 3 )

Tk mk nha

9 tháng 2 2018

a) (n mũ 2+n) chia hết cho 2 

=> n mũ 2 +n thuộc Ư(2), tự tìm ước của 2

9 tháng 2 2018

\(n^2+n=n\left(n+1\right)\)

Vì n(n+1) là tích 2 số nguyên liên tiếp nên chia hết cho 2 => đpcm

15 tháng 6 2017

a) Giải:

Đặt \(A_n=11^{n+2}+12^{2n+1}\)\((*)\) Với \(n=0\) ta có:

\(A_0=11^2+12^1=133\) \(⋮133\Rightarrow\) \((*)\) đúng

Giả sử \((*)\) đúng đến giá trị \(k=n\) tức là:

\(B_k=11^{k+2}+12^{2k+1}\) \(⋮133\left(1\right)\)

Xét \(B_{k+1}-B_k\)

\(=11^{k+1+2}+12^{2\left(k+1\right)+1}-\left(11^{k+2}+12^{2k+1}\right)\)

\(=11^{k+3}-11^{k+2}+12^{2k+3}-12^{2k+1}\)

\(=10.11^{k+2}+143.12^{2k+1}\)

\(=10.121.11^k+143.12.144^k\)

\(\equiv\) \(10.121.11^k+10.12.11^k\)

\(\equiv\) \(10.11^k\left(121+12\right)\) \(\equiv\) \(0\left(mod133\right)\)

Theo giả thiết quy nạy \(\left(1\right)\) ta có: \(B_k⋮133\Leftrightarrow B_{k+1}⋮133\)

Hay \((*)\) đúng với \(n=k+1\) \(\Rightarrow\) Đpcm

19 tháng 6 2017

Giải:

Ta có:

\(3^{4n+2}=9.9^{2n}=\) \(9.\left(17-8\right)^{2n}=17k+9.64^n\)

\(2.4^{3n+1}=8.64^n\)

\(\Rightarrow3^{4n+2}+2.4^{3n+1}=17k+17.64^n\)

\(=17\left(k+64^n\right)⋮17\forall x\in N\) (Đpcm)

11 tháng 2 2016

     A = ( 17^n + 1 )( 17^n + 2 )

=> A = ( 17^n x 17^n + 17^n )+( 2 x 17^n + 2 )

=> A = 17^n x 17^n + 17^n + 2 x 17^n  + 2 ( bỏ dấu ngoặc )

=> A= 17^n x 17^n +  ( 17^n +2 x 17^n ) +2

=> A= 17^n x 17^n + 3 x 17^n + 2

Mà 3 x 17^n chia hết cho 3

=> Tích A chia hết cho 3

22 tháng 7 2017

GIONG BAN TREN

26 tháng 6 2017

Với n = 0

\(\Rightarrow3.5^{2.0+1}+2^{3.0+1}=3.5+2=15+2=17⋮17\Rightarrow\)đúng với n = 0

Giả sử \(3.5^{2n+1}+2^{3n+1}\) đúng với n = k \(\in\) N*

\(\Rightarrow3.5^{2k+1}+2^{3k+1}⋮17\)

C/m : \(3.5^{2n+1}+2^{3n+1}\) đúng với n = k + 1 ( k \(\in\) N* )

Ta có :

\(3.5^{2n+1}+2^{3n+1}=3.5^{2\left(k+1\right)+1}+2^{3\left(k+1\right)+1}\)

\(=3.25.5^{2k+1}+8.3^{3k+1}=3.25.5^{2k+1}+25.2^{3k+1}-17.2^{3k+1}\)

\(=25\left(3.5^{2k+1}+2^{3k+1}\right)-17.2^{3k+1}\)

Vì : \(17.2^{3k+1}⋮17\) ; \(3.5^{2k+1}+2^{3k+1}⋮17\) theo phương pháp quy nạp

\(\Rightarrow3.5^{2\left(k+1\right)+1}+2^{3\left(k+1\right)+1}⋮17\)

Vậy ...